www.vorkurse.de
Ein Projekt von vorhilfe.de
Die Online-Kurse der Vorhilfe

E-Learning leicht gemacht.
Hallo Gast!einloggen | registrieren ]
Startseite · Mitglieder · Teams · Forum · Wissen · Kurse · Impressum
Forenbaum
^ Forenbaum
Status Mathe-Vorkurse
  Status Organisatorisches
  Status Schule
    Status Wiederholung Algebra
    Status Einführung Analysis
    Status Einführung Analytisc
    Status VK 21: Mathematik 6.
    Status VK 37: Kurvendiskussionen
    Status VK Abivorbereitungen
  Status Universität
    Status Lerngruppe LinAlg
    Status VK 13 Analysis I FH
    Status Algebra 2006
    Status VK 22: Algebra 2007
    Status GruMiHH 06
    Status VK 58: Algebra 1
    Status VK 59: Lineare Algebra
    Status VK 60: Analysis
    Status Wahrscheinlichkeitst

Gezeigt werden alle Foren bis zur Tiefe 2

Navigation
 Startseite...
 Neuerdings beta neu
 Forum...
 vorwissen...
 vorkurse...
 Werkzeuge...
 Nachhilfevermittlung beta...
 Online-Spiele beta
 Suchen
 Verein...
 Impressum
Das Projekt
Server und Internetanbindung werden durch Spenden finanziert.
Organisiert wird das Projekt von unserem Koordinatorenteam.
Hunderte Mitglieder helfen ehrenamtlich in unseren moderierten Foren.
Anbieter der Seite ist der gemeinnützige Verein "Vorhilfe.de e.V.".
Partnerseiten
Weitere Fächer:

Open Source FunktionenplotterFunkyPlot: Kostenloser und quelloffener Funktionenplotter für Linux und andere Betriebssysteme
Forum "Uni-Finanzmathematik" - Stochastisches Integral
Stochastisches Integral < Finanzmathematik < Finanz+Versicherung < Hochschule < Mathe < Vorhilfe
Ansicht: [ geschachtelt ] | ^ Forum "Uni-Finanzmathematik"  | ^^ Alle Foren  | ^ Forenbaum  | Materialien

Stochastisches Integral: Implementierung
Status: (Frage) beantwortet Status 
Datum: 01:47 Fr 11.03.2005
Autor: NY152

Hallo allerseits,
ich habe eine Frage bzgl. der Implemetierung eines Stochastisches Integrals.
Folgenden Ausdruck möchte ich gerne in VBA implementieren, welches aber ein stochastisches Integral enthält:

[mm] r_t = r_0*e^{-\kappa*t} + ( 1-e^{-\kappa*t})*\Phi + \sigma*e^{-\kappa*t}\int_0^t e^{\kappa u} dW_u [/mm]

Alle nötigen Parameter sind bekannt. Mir geht es in erster Linie darum, wie man im allgemeinen einen Stochastischen Integral (numerisch) implementiert.

Danke im Voraus.

Viele Grüße
Murat

Diese Frage habe ich in keinem anderen Forum gestellt.





        
Bezug
Stochastisches Integral: Antwort
Status: (Antwort) fertig Status 
Datum: 09:01 Fr 11.03.2005
Autor: Stefan

Hallo!

Dies implementiert man numerisch, dass man ein stochastisches Integral pfadweise als Riemann-Stieltjes-Integral auffasst (was natürlich mathematisch falsch ist, aber numerisch effektiv).

Dementsprechend wird [mm] $\int_0^t a(s)\, [/mm] ds + [mm] \int_0^t b(s)\, dW_s$ [/mm] durch die folgende Rekursion angenähert

[mm] $x_{j+1} [/mm] = [mm] x_j [/mm] + [mm] a(t_j) \Delta [/mm] t + [mm] b(t_j) [/mm] Z [mm] \sqrt{\Delta t}$, [/mm]

wobei $Z$ standardnormalverteilt ist (da musst du also stochastisch simulieren, also standardnormalverteilte Zufallszahlen einsetzen).

(Klar: Die Zuwächste [mm] $W_{t+\Delta t} [/mm] - [mm] W_t$ [/mm] sind ja gerade [mm] ${\cal N}(0,\sqrt{\Delta t})$ [/mm] verteilt. Das nutzt man hier aus.)

Damit bekommst du dann eine Realisierung deines stochastischen Integrals (was ja eine Zufallsvariable ist).

Genaueres findest du zum Beispiel in

Seydel: Einführung in die numerische Berechnung von Finanz-Derivaten, Springer-Verlag

oder auch

Kloeden, Platen: Numerical Solution of Stochastic Differential Equations, Springer-Verlag.

Liebe Grüße
Stefan


Bezug
                
Bezug
Stochastisches Integral: Frage (beantwortet)
Status: (Frage) beantwortet Status 
Datum: 00:03 Di 22.03.2005
Autor: NY152

Hallo Stefan,
vielen Dank für deine Antwort.

> Dies implementiert man numerisch, dass man ein
> stochastisches Integral pfadweise als
> Riemann-Stieltjes-Integral auffasst (was natürlich
> mathematisch falsch ist, aber numerisch effektiv).

   klar !

> Dementsprechend wird [mm]\int_0^t a(s)\, ds + \int_0^t b(s)\, dW_s[/mm]
> durch die folgende Rekursion angenähert
>  
> [mm]x_{j+1} = x_j + a(t_j) \Delta t + b(t_j) Z \sqrt{\Delta t}[/mm],

   unklar  ? Wie sieht dies denn in meinem Fall aus ?
   Also, $ [mm] r_t [/mm] = ... [mm] +\sigma\cdot{}e^{-\kappa\cdot{}t}\int_0^t e^{\kappa u} dW_u [/mm] $
    Leiter habe ich dies nicht verstanden.
> wobei [mm]Z[/mm] standardnormalverteilt ist (da musst du also

> stochastisch simulieren, also standardnormalverteilte
> Zufallszahlen einsetzen).

   klar! (Die Zufallszahlen werde ich mit der BoxMuller Methode erzeugen.)

> (Klar: Die Zuwächste [mm]W_{t+\Delta t} - W_t[/mm] sind ja gerade
> [mm]{\cal N}(0,\sqrt{\Delta t})[/mm] verteilt. Das nutzt man hier
> aus.)
>  
> Damit bekommst du dann eine Realisierung deines
> stochastischen Integrals (was ja eine Zufallsvariable
> ist).
>  
> Genaueres findest du zum Beispiel in
>  
> Seydel: Einführung in die numerische Berechnung von
> Finanz-Derivaten, Springer-Verlag

   (ist schwer zu bekommen)

>  
> oder auch
>  
> Kloeden, Platen: Numerical Solution of Stochastic
> Differential Equations, Springer-Verlag.
>  
> Liebe Grüße
>  Stefan
>  
>  


Viele Grüße
Murat

Bezug
                        
Bezug
Stochastisches Integral: Rückfrage
Status: (Mitteilung) Reaktion unnötig Status 
Datum: 02:23 Sa 26.03.2005
Autor: NY152

Hallo Stefan,
hab eine kurze Frage, ob ich dich richtig verstanden habe. Hast du bei deiner Antwort gemeint, daß man die SDE mit dem Euler - Algortihmus Implementiert bzw. löst ? Dies wendet man doch dann an, wenn man die SDE nicht analytisch lösen kann, oder ? Bei mir bekomme ich für das Vasicek - Modell eine analytische Lösung für [mm] $r_t$ [/mm] heraus. Die folgende SDE beschreibt das Modell von Vasicek:

[mm] $dr_t [/mm] = (b - a [mm] \cdot r_t) \cdot [/mm] dt + [mm] \sigma \cdot dW_t$ [/mm]

Die analytische Lösung für [mm] $r_t$ [/mm] laut nun:
[mm] $r_t [/mm] = [mm] r_0 \cdot e^{-a \cdot t} [/mm] + [mm] \frac{b}{a} \cdot [/mm] (1 - [mm] e^{-a \cdot t}) [/mm] + [mm] \sigma \cdot e^{-a \cdot t} \integral_0^t e^{a \cdot u} dW_u$ [/mm]

Mir geht es darum, wie ich den letzten Teil des Terms implementieren kann ?

> Dementsprechend wird [mm]\int_0^t a(s)\, ds + \int_0^t b(s)\, dW_s[/mm]
> durch die folgende Rekursion angenähert
>  
> [mm]x_{j+1} = x_j + a(t_j) \Delta t + b(t_j) Z \sqrt{\Delta t}[/mm],
>  
> wobei [mm]Z[/mm] standardnormalverteilt ist (da musst du also
> stochastisch simulieren, also standardnormalverteilte
> Zufallszahlen einsetzen).
>  
> (Klar: Die Zuwächste [mm]W_{t+\Delta t} - W_t[/mm] sind ja gerade
> [mm]{\cal N}(0,\sqrt{\Delta t})[/mm] verteilt. Das nutzt man hier
> aus.)
>  
> Damit bekommst du dann eine Realisierung deines
> stochastischen Integrals (was ja eine Zufallsvariable
> ist).
>  
> Genaueres findest du zum Beispiel in
>  
> Seydel: Einführung in die numerische Berechnung von
> Finanz-Derivaten, Springer-Verlag

   (hab es nun bekommen)

>  
> oder auch
>  
> Kloeden, Platen: Numerical Solution of Stochastic
> Differential Equations, Springer-Verlag.
>  
> Liebe Grüße
>  Stefan
>  
>  
>

Viele Grüße
Murat

Bezug
                                
Bezug
Stochastisches Integral: Mitteilung
Status: (Mitteilung) Reaktion unnötig Status 
Datum: 15:19 Mo 28.03.2005
Autor: Stefan

Hallo Murat!

Ja, ich kenne das Vasicek-Modell sehr gut und weiß, dass es eine analytische Lösung gibt. Dennoch simuliert man die Zufallsvariable wie von mir beschrieben, also so:

[mm]\int_0^t b(s)\, dW_s[/mm]

wird implementiert durch

[mm]x_0=0[/mm]
[mm]x_{j+1} = x_j + b(t_j) Z \sqrt{\Delta t}[/mm] [mm] ($j=0,1,\ldots,N-1$), [/mm]
[mm]x_N = \mbox{Näherungslösung für das stochastische Integral}[/mm],

wobei du $b$ in deinem Modell dann entsprechend ablesen musst. Dies meinte ich mit "pfadweises Auffassen als Riemann-Stieltjes-Integral".

Du diskretisiert also die Zeitachse und nutzt aus, dass die Zuwächse des Wiener-Prozesses normalverteilt sind.

Wenn du den Wiener-Prozess selbst implementierst, dann machst du es ja genauso, dass du die Zeitachse diskretisierst und normalverteilte Zufallszahlen einsetzt. Hier geht es eben fast genauso, nur dass man den Integranden (ausgewertet an einer Stelle) noch mitschleppen muss.

Viele Grüße
Stefan

Bezug
Ansicht: [ geschachtelt ] | ^ Forum "Uni-Finanzmathematik"  | ^^ Alle Foren  | ^ Forenbaum  | Materialien


^ Seitenanfang ^
www.vorkurse.de
[ Startseite | Mitglieder | Teams | Forum | Wissen | Kurse | Impressum ]